Prove that the sequence (1+1/n)^n is convergent [duplicate] Announcing the arrival of Valued Associate #679: Cesar Manara Planned maintenance scheduled April 23, 2019 at 00:00UTC (8:00pm US/Eastern)What is the most elementary proof that $lim_n to infty (1+1/n)^n$ exists?Is this proof that $lim_n to infty (1+1/n)^n$ exists (1) new, (2) interesting?Proving convergence of a sequence given by $a_n+1=A+Ba_n^3$Show that the sequence $x_n=left[1+frac(-1)^nnright]$ is convergent using monotone convergence theorem.Show that the sequence $x_n+1=x_n(2-x_n)$ is convergentIf $sum a_n$ is convergent and $a_n>0$, then $sum (-1)^n a_n$ is convergent.The Limit of The Sequence Given By a Recursive RelationLet $x_1 = 2$, $x_n+1=2+frac1x_n$, $n in mathbbN$. Does the sequence converge or diverge?Prove Bernoulli's inequality for irrationalsMonotone of a sequence without Bernouli's InequalityConvergent Sequence Summation ProblemAbout the proof that “ a bounded and monotone sequence is convergent”

How come Sam didn't become Lord of Horn Hill?

Central Vacuuming: Is it worth it, and how does it compare to normal vacuuming?

Converted a Scalar function to a TVF function for parallel execution-Still running in Serial mode

Selecting user stories during sprint planning

Do any jurisdictions seriously consider reclassifying social media websites as publishers?

How would a mousetrap for use in space work?

How can I reduce the gap between left and right of cdot with a macro?

How to write this math term? with cases it isn't working

Putting class ranking in CV, but against dept guidelines

How to play a character with a disability or mental disorder without being offensive?

Using audio cues to encourage good posture

Is grep documentation about ignoring case wrong, since it doesn't ignore case in filenames?

Why do early math courses focus on the cross sections of a cone and not on other 3D objects?

Drawing without replacement: why is the order of draw irrelevant?

SF book about people trapped in a series of worlds they imagine

Question about debouncing - delay of state change

Hangman Game with C++

Is there hard evidence that the grant peer review system performs significantly better than random?

Why weren't discrete x86 CPUs ever used in game hardware?

If Windows 7 doesn't support WSL, then what does Linux subsystem option mean?

Taylor expansion of ln(1-x)

Is it fair for a professor to grade us on the possession of past papers?

How to tell that you are a giant?

Why do we bend a book to keep it straight?



Prove that the sequence (1+1/n)^n is convergent [duplicate]



Announcing the arrival of Valued Associate #679: Cesar Manara
Planned maintenance scheduled April 23, 2019 at 00:00UTC (8:00pm US/Eastern)What is the most elementary proof that $lim_n to infty (1+1/n)^n$ exists?Is this proof that $lim_n to infty (1+1/n)^n$ exists (1) new, (2) interesting?Proving convergence of a sequence given by $a_n+1=A+Ba_n^3$Show that the sequence $x_n=left[1+frac(-1)^nnright]$ is convergent using monotone convergence theorem.Show that the sequence $x_n+1=x_n(2-x_n)$ is convergentIf $sum a_n$ is convergent and $a_n>0$, then $sum (-1)^n a_n$ is convergent.The Limit of The Sequence Given By a Recursive RelationLet $x_1 = 2$, $x_n+1=2+frac1x_n$, $n in mathbbN$. Does the sequence converge or diverge?Prove Bernoulli's inequality for irrationalsMonotone of a sequence without Bernouli's InequalityConvergent Sequence Summation ProblemAbout the proof that “ a bounded and monotone sequence is convergent”










0












$begingroup$



This question already has an answer here:



  • What is the most elementary proof that $lim_n to infty (1+1/n)^n$ exists?

    3 answers



I know the proof using binomial expansion and then by monotone convergence theorem.
But i want to collect some other proofs without using the binomial expansion.



*if you could provide the answer without the concept of calculus it will be appreciated *
But any way do provide whatever method you can.










share|cite|improve this question











$endgroup$



marked as duplicate by Somos, Sil, YiFan, Shailesh, Eevee Trainer Mar 28 at 0:15


This question has been asked before and already has an answer. If those answers do not fully address your question, please ask a new question.













  • 1




    $begingroup$
    It is hard to avoid "the concept of calculus" since limits and convergent sequences are a part of that concept. On the other hand, it would help to specify what tools you're happy with using, since this result is used in developing some of them. (For example, if you define $e^x = lim_n to infty (1 + x/n)^n$, then clearly we should not be using $e^x$ in the process of showing that this limit exists.)
    $endgroup$
    – Misha Lavrov
    Mar 27 at 17:24










  • $begingroup$
    math.stackexchange.com/questions/389793/…
    $endgroup$
    – marty cohen
    Mar 27 at 17:56















0












$begingroup$



This question already has an answer here:



  • What is the most elementary proof that $lim_n to infty (1+1/n)^n$ exists?

    3 answers



I know the proof using binomial expansion and then by monotone convergence theorem.
But i want to collect some other proofs without using the binomial expansion.



*if you could provide the answer without the concept of calculus it will be appreciated *
But any way do provide whatever method you can.










share|cite|improve this question











$endgroup$



marked as duplicate by Somos, Sil, YiFan, Shailesh, Eevee Trainer Mar 28 at 0:15


This question has been asked before and already has an answer. If those answers do not fully address your question, please ask a new question.













  • 1




    $begingroup$
    It is hard to avoid "the concept of calculus" since limits and convergent sequences are a part of that concept. On the other hand, it would help to specify what tools you're happy with using, since this result is used in developing some of them. (For example, if you define $e^x = lim_n to infty (1 + x/n)^n$, then clearly we should not be using $e^x$ in the process of showing that this limit exists.)
    $endgroup$
    – Misha Lavrov
    Mar 27 at 17:24










  • $begingroup$
    math.stackexchange.com/questions/389793/…
    $endgroup$
    – marty cohen
    Mar 27 at 17:56













0












0








0





$begingroup$



This question already has an answer here:



  • What is the most elementary proof that $lim_n to infty (1+1/n)^n$ exists?

    3 answers



I know the proof using binomial expansion and then by monotone convergence theorem.
But i want to collect some other proofs without using the binomial expansion.



*if you could provide the answer without the concept of calculus it will be appreciated *
But any way do provide whatever method you can.










share|cite|improve this question











$endgroup$





This question already has an answer here:



  • What is the most elementary proof that $lim_n to infty (1+1/n)^n$ exists?

    3 answers



I know the proof using binomial expansion and then by monotone convergence theorem.
But i want to collect some other proofs without using the binomial expansion.



*if you could provide the answer without the concept of calculus it will be appreciated *
But any way do provide whatever method you can.





This question already has an answer here:



  • What is the most elementary proof that $lim_n to infty (1+1/n)^n$ exists?

    3 answers







sequences-and-series






share|cite|improve this question















share|cite|improve this question













share|cite|improve this question




share|cite|improve this question








edited Mar 27 at 17:19







risav jain

















asked Mar 27 at 17:05









risav jainrisav jain

94




94




marked as duplicate by Somos, Sil, YiFan, Shailesh, Eevee Trainer Mar 28 at 0:15


This question has been asked before and already has an answer. If those answers do not fully address your question, please ask a new question.









marked as duplicate by Somos, Sil, YiFan, Shailesh, Eevee Trainer Mar 28 at 0:15


This question has been asked before and already has an answer. If those answers do not fully address your question, please ask a new question.









  • 1




    $begingroup$
    It is hard to avoid "the concept of calculus" since limits and convergent sequences are a part of that concept. On the other hand, it would help to specify what tools you're happy with using, since this result is used in developing some of them. (For example, if you define $e^x = lim_n to infty (1 + x/n)^n$, then clearly we should not be using $e^x$ in the process of showing that this limit exists.)
    $endgroup$
    – Misha Lavrov
    Mar 27 at 17:24










  • $begingroup$
    math.stackexchange.com/questions/389793/…
    $endgroup$
    – marty cohen
    Mar 27 at 17:56












  • 1




    $begingroup$
    It is hard to avoid "the concept of calculus" since limits and convergent sequences are a part of that concept. On the other hand, it would help to specify what tools you're happy with using, since this result is used in developing some of them. (For example, if you define $e^x = lim_n to infty (1 + x/n)^n$, then clearly we should not be using $e^x$ in the process of showing that this limit exists.)
    $endgroup$
    – Misha Lavrov
    Mar 27 at 17:24










  • $begingroup$
    math.stackexchange.com/questions/389793/…
    $endgroup$
    – marty cohen
    Mar 27 at 17:56







1




1




$begingroup$
It is hard to avoid "the concept of calculus" since limits and convergent sequences are a part of that concept. On the other hand, it would help to specify what tools you're happy with using, since this result is used in developing some of them. (For example, if you define $e^x = lim_n to infty (1 + x/n)^n$, then clearly we should not be using $e^x$ in the process of showing that this limit exists.)
$endgroup$
– Misha Lavrov
Mar 27 at 17:24




$begingroup$
It is hard to avoid "the concept of calculus" since limits and convergent sequences are a part of that concept. On the other hand, it would help to specify what tools you're happy with using, since this result is used in developing some of them. (For example, if you define $e^x = lim_n to infty (1 + x/n)^n$, then clearly we should not be using $e^x$ in the process of showing that this limit exists.)
$endgroup$
– Misha Lavrov
Mar 27 at 17:24












$begingroup$
math.stackexchange.com/questions/389793/…
$endgroup$
– marty cohen
Mar 27 at 17:56




$begingroup$
math.stackexchange.com/questions/389793/…
$endgroup$
– marty cohen
Mar 27 at 17:56










3 Answers
3






active

oldest

votes


















0












$begingroup$

If $a_n = left(1 + frac1nright)^n$, then $ln(a_n) = n cdot lnleft(1+frac1nright)$. This is indeterminate, but you can apply l'Hopital's rule to find $ln(a_n)to 1$ with $n$, so that, using the fact that continuous functions preserve limits, $a_nto e$.






share|cite|improve this answer









$endgroup$












  • $begingroup$
    Is their a way without using the concept of calculus?
    $endgroup$
    – risav jain
    Mar 27 at 17:16


















0












$begingroup$

Let $a_n$ the given sequence. We have



$$w_n:=ln(a_n+1)-ln a_n=lnfraca_n+1a_n=(n+1)ln(1+1/(n+1))-nln(1+1/n)=O(frac1n^2)$$
so the series $sum w_n$ is convergent and so by telescoping the sequence $(ln(a_n))$ is convergent. Conclude using the exponential function.






share|cite|improve this answer









$endgroup$




















    0












    $begingroup$

    You can do this in two steps; here, no calculus is needed:



    Let $a_n=(1+frac1n)^n$.



    1. Show that $a_nleqsum_k=0^nfrac1k!$ for all $ninmathbb N$.

    2. Show that $a_n$ is monotonically increasing.

    To show (2), you have to prove that $fraca_n+1a_ngeq1$; some calculations and Bernoulli's inequality are involved here:



    $$fraca_n+1a_n=fracleft(1+frac11+nright)^n+1left(1+frac1nright)^n=left(1+frac1nright)left(frac1+frac11+n1+frac1nright)^n+1=left(1+frac1nright)left(fracn^2+2nn^2+2n+1right)^n+1=left(1+frac1nright)left(1-frac1n^2+2n+1right)^n+1geqleft(1+frac1nright)left(1-fracn+1n^2+2n+1right)=fracn+1nfracnn+1=1$$



    Because the series on the right hand side in (1) is convergent, $(a_n)$ is bounded. Together with (2), this implies that the sequence converges.






    share|cite|improve this answer











    $endgroup$












    • $begingroup$
      1 is easy, 2 is not.
      $endgroup$
      – marty cohen
      Mar 27 at 17:55










    • $begingroup$
      Edited my answer to include the calculations for (2).
      $endgroup$
      – st.math
      Mar 27 at 18:08










    • $begingroup$
      Third line, second expression should be 1-, not 1+, so it's $1-1/(n+1) = n/(n+1)$ so you get 1.
      $endgroup$
      – marty cohen
      Mar 27 at 21:09










    • $begingroup$
      See this question of mine which does essentially the same thing in more detail: math.stackexchange.com/questions/1387272/…
      $endgroup$
      – marty cohen
      Mar 27 at 21:13










    • $begingroup$
      You are right, thanks. :) Corrected it.
      $endgroup$
      – st.math
      Mar 27 at 22:02

















    3 Answers
    3






    active

    oldest

    votes








    3 Answers
    3






    active

    oldest

    votes









    active

    oldest

    votes






    active

    oldest

    votes









    0












    $begingroup$

    If $a_n = left(1 + frac1nright)^n$, then $ln(a_n) = n cdot lnleft(1+frac1nright)$. This is indeterminate, but you can apply l'Hopital's rule to find $ln(a_n)to 1$ with $n$, so that, using the fact that continuous functions preserve limits, $a_nto e$.






    share|cite|improve this answer









    $endgroup$












    • $begingroup$
      Is their a way without using the concept of calculus?
      $endgroup$
      – risav jain
      Mar 27 at 17:16















    0












    $begingroup$

    If $a_n = left(1 + frac1nright)^n$, then $ln(a_n) = n cdot lnleft(1+frac1nright)$. This is indeterminate, but you can apply l'Hopital's rule to find $ln(a_n)to 1$ with $n$, so that, using the fact that continuous functions preserve limits, $a_nto e$.






    share|cite|improve this answer









    $endgroup$












    • $begingroup$
      Is their a way without using the concept of calculus?
      $endgroup$
      – risav jain
      Mar 27 at 17:16













    0












    0








    0





    $begingroup$

    If $a_n = left(1 + frac1nright)^n$, then $ln(a_n) = n cdot lnleft(1+frac1nright)$. This is indeterminate, but you can apply l'Hopital's rule to find $ln(a_n)to 1$ with $n$, so that, using the fact that continuous functions preserve limits, $a_nto e$.






    share|cite|improve this answer









    $endgroup$



    If $a_n = left(1 + frac1nright)^n$, then $ln(a_n) = n cdot lnleft(1+frac1nright)$. This is indeterminate, but you can apply l'Hopital's rule to find $ln(a_n)to 1$ with $n$, so that, using the fact that continuous functions preserve limits, $a_nto e$.







    share|cite|improve this answer












    share|cite|improve this answer



    share|cite|improve this answer










    answered Mar 27 at 17:14









    MatthewPeterMatthewPeter

    191




    191











    • $begingroup$
      Is their a way without using the concept of calculus?
      $endgroup$
      – risav jain
      Mar 27 at 17:16
















    • $begingroup$
      Is their a way without using the concept of calculus?
      $endgroup$
      – risav jain
      Mar 27 at 17:16















    $begingroup$
    Is their a way without using the concept of calculus?
    $endgroup$
    – risav jain
    Mar 27 at 17:16




    $begingroup$
    Is their a way without using the concept of calculus?
    $endgroup$
    – risav jain
    Mar 27 at 17:16











    0












    $begingroup$

    Let $a_n$ the given sequence. We have



    $$w_n:=ln(a_n+1)-ln a_n=lnfraca_n+1a_n=(n+1)ln(1+1/(n+1))-nln(1+1/n)=O(frac1n^2)$$
    so the series $sum w_n$ is convergent and so by telescoping the sequence $(ln(a_n))$ is convergent. Conclude using the exponential function.






    share|cite|improve this answer









    $endgroup$

















      0












      $begingroup$

      Let $a_n$ the given sequence. We have



      $$w_n:=ln(a_n+1)-ln a_n=lnfraca_n+1a_n=(n+1)ln(1+1/(n+1))-nln(1+1/n)=O(frac1n^2)$$
      so the series $sum w_n$ is convergent and so by telescoping the sequence $(ln(a_n))$ is convergent. Conclude using the exponential function.






      share|cite|improve this answer









      $endgroup$















        0












        0








        0





        $begingroup$

        Let $a_n$ the given sequence. We have



        $$w_n:=ln(a_n+1)-ln a_n=lnfraca_n+1a_n=(n+1)ln(1+1/(n+1))-nln(1+1/n)=O(frac1n^2)$$
        so the series $sum w_n$ is convergent and so by telescoping the sequence $(ln(a_n))$ is convergent. Conclude using the exponential function.






        share|cite|improve this answer









        $endgroup$



        Let $a_n$ the given sequence. We have



        $$w_n:=ln(a_n+1)-ln a_n=lnfraca_n+1a_n=(n+1)ln(1+1/(n+1))-nln(1+1/n)=O(frac1n^2)$$
        so the series $sum w_n$ is convergent and so by telescoping the sequence $(ln(a_n))$ is convergent. Conclude using the exponential function.







        share|cite|improve this answer












        share|cite|improve this answer



        share|cite|improve this answer










        answered Mar 27 at 17:23









        user296113user296113

        7,015928




        7,015928





















            0












            $begingroup$

            You can do this in two steps; here, no calculus is needed:



            Let $a_n=(1+frac1n)^n$.



            1. Show that $a_nleqsum_k=0^nfrac1k!$ for all $ninmathbb N$.

            2. Show that $a_n$ is monotonically increasing.

            To show (2), you have to prove that $fraca_n+1a_ngeq1$; some calculations and Bernoulli's inequality are involved here:



            $$fraca_n+1a_n=fracleft(1+frac11+nright)^n+1left(1+frac1nright)^n=left(1+frac1nright)left(frac1+frac11+n1+frac1nright)^n+1=left(1+frac1nright)left(fracn^2+2nn^2+2n+1right)^n+1=left(1+frac1nright)left(1-frac1n^2+2n+1right)^n+1geqleft(1+frac1nright)left(1-fracn+1n^2+2n+1right)=fracn+1nfracnn+1=1$$



            Because the series on the right hand side in (1) is convergent, $(a_n)$ is bounded. Together with (2), this implies that the sequence converges.






            share|cite|improve this answer











            $endgroup$












            • $begingroup$
              1 is easy, 2 is not.
              $endgroup$
              – marty cohen
              Mar 27 at 17:55










            • $begingroup$
              Edited my answer to include the calculations for (2).
              $endgroup$
              – st.math
              Mar 27 at 18:08










            • $begingroup$
              Third line, second expression should be 1-, not 1+, so it's $1-1/(n+1) = n/(n+1)$ so you get 1.
              $endgroup$
              – marty cohen
              Mar 27 at 21:09










            • $begingroup$
              See this question of mine which does essentially the same thing in more detail: math.stackexchange.com/questions/1387272/…
              $endgroup$
              – marty cohen
              Mar 27 at 21:13










            • $begingroup$
              You are right, thanks. :) Corrected it.
              $endgroup$
              – st.math
              Mar 27 at 22:02















            0












            $begingroup$

            You can do this in two steps; here, no calculus is needed:



            Let $a_n=(1+frac1n)^n$.



            1. Show that $a_nleqsum_k=0^nfrac1k!$ for all $ninmathbb N$.

            2. Show that $a_n$ is monotonically increasing.

            To show (2), you have to prove that $fraca_n+1a_ngeq1$; some calculations and Bernoulli's inequality are involved here:



            $$fraca_n+1a_n=fracleft(1+frac11+nright)^n+1left(1+frac1nright)^n=left(1+frac1nright)left(frac1+frac11+n1+frac1nright)^n+1=left(1+frac1nright)left(fracn^2+2nn^2+2n+1right)^n+1=left(1+frac1nright)left(1-frac1n^2+2n+1right)^n+1geqleft(1+frac1nright)left(1-fracn+1n^2+2n+1right)=fracn+1nfracnn+1=1$$



            Because the series on the right hand side in (1) is convergent, $(a_n)$ is bounded. Together with (2), this implies that the sequence converges.






            share|cite|improve this answer











            $endgroup$












            • $begingroup$
              1 is easy, 2 is not.
              $endgroup$
              – marty cohen
              Mar 27 at 17:55










            • $begingroup$
              Edited my answer to include the calculations for (2).
              $endgroup$
              – st.math
              Mar 27 at 18:08










            • $begingroup$
              Third line, second expression should be 1-, not 1+, so it's $1-1/(n+1) = n/(n+1)$ so you get 1.
              $endgroup$
              – marty cohen
              Mar 27 at 21:09










            • $begingroup$
              See this question of mine which does essentially the same thing in more detail: math.stackexchange.com/questions/1387272/…
              $endgroup$
              – marty cohen
              Mar 27 at 21:13










            • $begingroup$
              You are right, thanks. :) Corrected it.
              $endgroup$
              – st.math
              Mar 27 at 22:02













            0












            0








            0





            $begingroup$

            You can do this in two steps; here, no calculus is needed:



            Let $a_n=(1+frac1n)^n$.



            1. Show that $a_nleqsum_k=0^nfrac1k!$ for all $ninmathbb N$.

            2. Show that $a_n$ is monotonically increasing.

            To show (2), you have to prove that $fraca_n+1a_ngeq1$; some calculations and Bernoulli's inequality are involved here:



            $$fraca_n+1a_n=fracleft(1+frac11+nright)^n+1left(1+frac1nright)^n=left(1+frac1nright)left(frac1+frac11+n1+frac1nright)^n+1=left(1+frac1nright)left(fracn^2+2nn^2+2n+1right)^n+1=left(1+frac1nright)left(1-frac1n^2+2n+1right)^n+1geqleft(1+frac1nright)left(1-fracn+1n^2+2n+1right)=fracn+1nfracnn+1=1$$



            Because the series on the right hand side in (1) is convergent, $(a_n)$ is bounded. Together with (2), this implies that the sequence converges.






            share|cite|improve this answer











            $endgroup$



            You can do this in two steps; here, no calculus is needed:



            Let $a_n=(1+frac1n)^n$.



            1. Show that $a_nleqsum_k=0^nfrac1k!$ for all $ninmathbb N$.

            2. Show that $a_n$ is monotonically increasing.

            To show (2), you have to prove that $fraca_n+1a_ngeq1$; some calculations and Bernoulli's inequality are involved here:



            $$fraca_n+1a_n=fracleft(1+frac11+nright)^n+1left(1+frac1nright)^n=left(1+frac1nright)left(frac1+frac11+n1+frac1nright)^n+1=left(1+frac1nright)left(fracn^2+2nn^2+2n+1right)^n+1=left(1+frac1nright)left(1-frac1n^2+2n+1right)^n+1geqleft(1+frac1nright)left(1-fracn+1n^2+2n+1right)=fracn+1nfracnn+1=1$$



            Because the series on the right hand side in (1) is convergent, $(a_n)$ is bounded. Together with (2), this implies that the sequence converges.







            share|cite|improve this answer














            share|cite|improve this answer



            share|cite|improve this answer








            edited Mar 27 at 21:55

























            answered Mar 27 at 17:27









            st.mathst.math

            1,268115




            1,268115











            • $begingroup$
              1 is easy, 2 is not.
              $endgroup$
              – marty cohen
              Mar 27 at 17:55










            • $begingroup$
              Edited my answer to include the calculations for (2).
              $endgroup$
              – st.math
              Mar 27 at 18:08










            • $begingroup$
              Third line, second expression should be 1-, not 1+, so it's $1-1/(n+1) = n/(n+1)$ so you get 1.
              $endgroup$
              – marty cohen
              Mar 27 at 21:09










            • $begingroup$
              See this question of mine which does essentially the same thing in more detail: math.stackexchange.com/questions/1387272/…
              $endgroup$
              – marty cohen
              Mar 27 at 21:13










            • $begingroup$
              You are right, thanks. :) Corrected it.
              $endgroup$
              – st.math
              Mar 27 at 22:02
















            • $begingroup$
              1 is easy, 2 is not.
              $endgroup$
              – marty cohen
              Mar 27 at 17:55










            • $begingroup$
              Edited my answer to include the calculations for (2).
              $endgroup$
              – st.math
              Mar 27 at 18:08










            • $begingroup$
              Third line, second expression should be 1-, not 1+, so it's $1-1/(n+1) = n/(n+1)$ so you get 1.
              $endgroup$
              – marty cohen
              Mar 27 at 21:09










            • $begingroup$
              See this question of mine which does essentially the same thing in more detail: math.stackexchange.com/questions/1387272/…
              $endgroup$
              – marty cohen
              Mar 27 at 21:13










            • $begingroup$
              You are right, thanks. :) Corrected it.
              $endgroup$
              – st.math
              Mar 27 at 22:02















            $begingroup$
            1 is easy, 2 is not.
            $endgroup$
            – marty cohen
            Mar 27 at 17:55




            $begingroup$
            1 is easy, 2 is not.
            $endgroup$
            – marty cohen
            Mar 27 at 17:55












            $begingroup$
            Edited my answer to include the calculations for (2).
            $endgroup$
            – st.math
            Mar 27 at 18:08




            $begingroup$
            Edited my answer to include the calculations for (2).
            $endgroup$
            – st.math
            Mar 27 at 18:08












            $begingroup$
            Third line, second expression should be 1-, not 1+, so it's $1-1/(n+1) = n/(n+1)$ so you get 1.
            $endgroup$
            – marty cohen
            Mar 27 at 21:09




            $begingroup$
            Third line, second expression should be 1-, not 1+, so it's $1-1/(n+1) = n/(n+1)$ so you get 1.
            $endgroup$
            – marty cohen
            Mar 27 at 21:09












            $begingroup$
            See this question of mine which does essentially the same thing in more detail: math.stackexchange.com/questions/1387272/…
            $endgroup$
            – marty cohen
            Mar 27 at 21:13




            $begingroup$
            See this question of mine which does essentially the same thing in more detail: math.stackexchange.com/questions/1387272/…
            $endgroup$
            – marty cohen
            Mar 27 at 21:13












            $begingroup$
            You are right, thanks. :) Corrected it.
            $endgroup$
            – st.math
            Mar 27 at 22:02




            $begingroup$
            You are right, thanks. :) Corrected it.
            $endgroup$
            – st.math
            Mar 27 at 22:02



            Popular posts from this blog

            Lowndes Grove History Architecture References Navigation menu32°48′6″N 79°57′58″W / 32.80167°N 79.96611°W / 32.80167; -79.9661132°48′6″N 79°57′58″W / 32.80167°N 79.96611°W / 32.80167; -79.9661178002500"National Register Information System"Historic houses of South Carolina"Lowndes Grove""+32° 48' 6.00", −79° 57' 58.00""Lowndes Grove, Charleston County (260 St. Margaret St., Charleston)""Lowndes Grove"The Charleston ExpositionIt Happened in South Carolina"Lowndes Grove (House), Saint Margaret Street & Sixth Avenue, Charleston, Charleston County, SC(Photographs)"Plantations of the Carolina Low Countrye

            random experiment with two different functions on unit interval Announcing the arrival of Valued Associate #679: Cesar Manara Planned maintenance scheduled April 23, 2019 at 00:00UTC (8:00pm US/Eastern)Random variable and probability space notionsRandom Walk with EdgesFinding functions where the increase over a random interval is Poisson distributedNumber of days until dayCan an observed event in fact be of zero probability?Unit random processmodels of coins and uniform distributionHow to get the number of successes given $n$ trials , probability $P$ and a random variable $X$Absorbing Markov chain in a computer. Is “almost every” turned into always convergence in computer executions?Stopped random walk is not uniformly integrable

            How should I support this large drywall patch? Planned maintenance scheduled April 23, 2019 at 00:00UTC (8:00pm US/Eastern) Announcing the arrival of Valued Associate #679: Cesar Manara Unicorn Meta Zoo #1: Why another podcast?How do I cover large gaps in drywall?How do I keep drywall around a patch from crumbling?Can I glue a second layer of drywall?How to patch long strip on drywall?Large drywall patch: how to avoid bulging seams?Drywall Mesh Patch vs. Bulge? To remove or not to remove?How to fix this drywall job?Prep drywall before backsplashWhat's the best way to fix this horrible drywall patch job?Drywall patching using 3M Patch Plus Primer